10
$\begingroup$

Let $t_{1},t_{2},\ldots, t_{n}$ be i.i.d. real Gaussian random variables of zero mean and variance one. Let $a_{1},a_{2},\ldots, a_{n}$ be positive and fixed real numbers and define the random polynomial $$ p(z):=\sum_{k=1}^{n}{a_{k}t_{k}z^{k}}. $$

Define the random variable $$ m=\max_{|z|=1}\Big\{\mathrm{Re}(p(z))\Big\}=\max_{\theta\in (0,2\pi]}\Bigg\{\sum_{k=1}^{n}{a_{k}t_{k}\cos(k\theta)}\Bigg\}. $$

How can one compute the probability distribution of $m$? Can we compute at least the first few moments $\mathbb{E}(m)$ and $\mathbb{E}(m^2)$?

$\endgroup$
4
  • $\begingroup$ Have you taken a look at Marcus and Pisier's book? $\endgroup$ Apr 23, 2011 at 15:38
  • $\begingroup$ No, I'm not familiar with that book. Do you think it can help? $\endgroup$
    – ght
    Apr 23, 2011 at 16:21
  • 1
    $\begingroup$ Yes. The sharpest known results on what you ask are probably in there; the main difficulty might be extracting the rather classical special case you're interested in. You could also take a look at Kahane's book, Some Random Series of Functions. $\endgroup$ Apr 24, 2011 at 13:37
  • $\begingroup$ This question looks very interesting! Although so many years have passed, may I ask did you finally find an answer to this question? $\endgroup$
    – MikeG
    Nov 24, 2022 at 8:46

2 Answers 2

5
$\begingroup$

By coincidence I recently needed to consider essentially this same question.

By homogeneity one can fix the value of $\sum_{k=1}^{n} a_k^2$. Let us take this to be $n$. If all of the a_i's were equal to 1, then the max m will be asymptotically $\sqrt{n \ln(n)}$ (with an error term of $O\left( \sqrt{\frac{n}{\ln(n)}} \ln \ln(n) \right)$) with probability $1$. In the case of random +/- signs, the upper-bound is an old result of Salem and Zygmund (Some properties of trigonometric series whose terms have random signs) and the lower bound is a result of Halasz (On a result of Salem and Zygmund concerning random polynomials). The proof should easily modify to include the case of random Gaussians or more general i.i.d. random variables.

Its more difficult to state a result for general coefficients, since if the coefficients are concentrated on just a few terms (say just one term) the above result is obviously false. However, as long as not too much of the $\ell^2$ weight is concentrated on a single coefficient then the above result should still hold. Results in this direction are contained in the Salem and Zygmund paper.

$\endgroup$
6
  • $\begingroup$ @Mark: Thanks. As a matter of fact I'm interested in the case $a_k=\frac{1}{k}$ what paper should I looked? $\endgroup$
    – ght
    Apr 22, 2011 at 22:54
  • $\begingroup$ By homogeneity, the max in this case should asymptotically be $\frac{\ln(n)}{\sqrt{n}}$ almost surely. This is done in the paper Halász, G. On a result of Salem and Zygmund concerning random polynomials. Studia Sci. Math. Hungar. 8 (1973), 369–377, when the rv's are random signs. It should be straightforward to modify the argument for Gaussians. $\endgroup$
    – Mark Lewko
    Apr 22, 2011 at 23:02
  • $\begingroup$ Of course I meant $\sqrt{\frac{\ln(n)}{n}}$. $\endgroup$
    – Mark Lewko
    Apr 22, 2011 at 23:16
  • $\begingroup$ Did you mean to say $\sqrt{\frac{\ln(n)}{n}}$? It seems to me that the max cannot go to zero as $n\to\infty$. Other thing, I believe that these papers deal with the case $$ \max_{\theta\in[0,2\pi]}{|\sum_{k=1}^{n}{a_k t_k \cos(k\theta)}|} $$ which is different to what I'm asking. $\endgroup$
    – ght
    Apr 23, 2011 at 1:06
  • $\begingroup$ Sorry, I had misread your statement $a_k =1/k$ as $a_k=1/n$ (so $\sqrt{\frac{ln(n)}{n}}$ won't be right in your case). $\endgroup$
    – Mark Lewko
    Apr 23, 2011 at 1:08
2
$\begingroup$

The case of $ a_k=1/k$ and Gaussian coefficents is easily seen that the series is a.s. convergent to a continuous function. (Corollary to Salem-Zygmund, already mentioned) So, in the case you are considering the distribution $m$ will have subgaussian tails.

Under weaker conditions on the iid random coefficents, and $ a_k=1/k$, this is discussed in an article of Michel Talagrand, A borderline Fourier Series" Ann Prob 1995. http://www.jstor.org/pss/2245006

$\endgroup$
1
  • $\begingroup$ I'm familiar with the paper that you mentioned where it is essentially proved that the random series $$ \sum_{n=1}^{\infty}{\frac{t_k}{k}\cos(k\theta)} $$ is absolutely convergent. How this helps to solve my problem? I'm considering only a finite sum (trigonometric polynomial). $\endgroup$
    – ght
    Apr 23, 2011 at 11:29

Your Answer

By clicking “Post Your Answer”, you agree to our terms of service and acknowledge you have read our privacy policy.

Not the answer you're looking for? Browse other questions tagged or ask your own question.